matheraum.de
Raum für Mathematik
Offene Informations- und Nachhilfegemeinschaft

Für Schüler, Studenten, Lehrer, Mathematik-Interessierte.
Hallo Gast!einloggen | registrieren ]
Startseite · Forum · Wissen · Kurse · Mitglieder · Team · Impressum
Forenbaum
^ Forenbaum
Status Schulmathe
  Status Primarstufe
  Status Mathe Klassen 5-7
  Status Mathe Klassen 8-10
  Status Oberstufenmathe
    Status Schul-Analysis
    Status Lin. Algebra/Vektor
    Status Stochastik
    Status Abivorbereitung
  Status Mathe-Wettbewerbe
    Status Bundeswettb. Mathe
    Status Deutsche MO
    Status Internationale MO
    Status MO andere Länder
    Status Känguru
  Status Sonstiges

Gezeigt werden alle Foren bis zur Tiefe 2

Navigation
 Startseite...
 Neuerdings beta neu
 Forum...
 vorwissen...
 vorkurse...
 Werkzeuge...
 Nachhilfevermittlung beta...
 Online-Spiele beta
 Suchen
 Verein...
 Impressum
Das Projekt
Server und Internetanbindung werden durch Spenden finanziert.
Organisiert wird das Projekt von unserem Koordinatorenteam.
Hunderte Mitglieder helfen ehrenamtlich in unseren moderierten Foren.
Anbieter der Seite ist der gemeinnützige Verein "Vorhilfe.de e.V.".
Partnerseiten
Weitere Fächer:

Open Source FunktionenplotterFunkyPlot: Kostenloser und quelloffener Funktionenplotter für Linux und andere Betriebssysteme
StartseiteMatheForenGewöhnliche DifferentialgleichungenBernoulli, getrennte Veränderl
Foren für weitere Schulfächer findest Du auf www.vorhilfe.de z.B. Informatik • Physik • Technik • Biologie • Chemie
Forum "Gewöhnliche Differentialgleichungen" - Bernoulli, getrennte Veränderl
Bernoulli, getrennte Veränderl < gewöhnliche < Differentialgl. < Analysis < Hochschule < Mathe < Vorhilfe
Ansicht: [ geschachtelt ] | ^ Forum "Gewöhnliche Differentialgleichungen"  | ^^ Alle Foren  | ^ Forenbaum  | Materialien

Bernoulli, getrennte Veränderl: Rückfrage, Idee
Status: (Frage) beantwortet Status 
Datum: 11:48 Mi 14.07.2010
Autor: Blinkly

Aufgabe
Bestimmen Sie die Lösung von
$$y' = (y + [mm] y^3) \cdot \sin(x), [/mm] y(0)=1$$
Lösen Sie die Dgl. sowohl als Typ "getrennte Veränderliche" wie auch als Bernoulli-DGL.

Hallo =)

Erst einmal vorweg:
Ich habe diese Frage in keinem Forum auf anderen Internetseiten gestellt ;-)

Habe die Aufgabe versucht zu lösen (auf beide Arten, mehr oder weniger erfolgreich) und habe leider noch ein paar Fragen, aber erstmal zu meiner Rechnung:

Bernoulli:
$y' = [mm] \sin(x)y [/mm] + [mm] sin(x)y^3, \alpha [/mm] = 3$
$u = [mm] y^{-2}$ [/mm]
$u' = [mm] -2\sin(x)u [/mm] - [mm] 2\sin(x)$ [/mm]
[mm] $u_h [/mm] = [mm] e^{2\cos(x)}$ [/mm]
[mm] $u_p [/mm] = [mm] e^{2\cos(x)} \cdot \int \frac{-2\sin(x)}{e^{2\cos(x)}} [/mm] dx = [mm] e^{2\cos(x)}\cdot -e^{-2\cos(x)} [/mm] = -1$ Substitution: $t = [mm] \cos(x)$ [/mm]
[mm] $\Rightarrow [/mm] u = c [mm] \cdot e^{2\cos(x)} [/mm] - 1$
$y = [mm] \frac{1}{\sqrt{u}}$ [/mm]
$y = [mm] \frac{1}{\sqrt{c\cdot e^{2\cos(x)}-1}}$ [/mm]
$c = [mm] \frac{2}{e^2}$ [/mm] mit $y(0) = 1$
[mm] $\Rightarrow [/mm] y = [mm] \frac{1}{\sqrt{2\cdot e^{2(\cos(x)-1)}-1}}$ [/mm]

Hier ist die Frage noch relativ einfach, stimmt das? :)


getrennte Veränderliche:
[mm] $\int\frac{1}{y+y^3} [/mm] dy = [mm] \int\sin(x)dx [/mm] + c$
[mm] $\int\frac{1}{y}dy [/mm] - [mm] \int\frac{y}{y^2+1} [/mm] dy = [mm] -\cos(x) [/mm] + c$
[mm] $\Rightarrow \ln|y| [/mm] - [mm] \frac{ln|y^2+1|}{2} [/mm] = [mm] -\cos(x) [/mm] + c$

Und das nun nach y aufzulösen bereitet mir Probleme, sofern ich überhaupt richtig gerechnet hab bis hier hin ;-)

Möglicher Ansatz von mir:
[mm] $\frac{1}{y^2} [/mm] + [mm] y^2 [/mm] + 1 = [mm] e^{2\cos(x)} [/mm] - [mm] e^{2c}$ [/mm]

Mehr hab ich dazu leider noch nicht =/

Ich bedanke mich schon mal für Eure Mühe =)

Grüße
Blinkly

        
Bezug
Bernoulli, getrennte Veränderl: Antwort
Status: (Antwort) fertig Status 
Datum: 12:59 Mi 14.07.2010
Autor: schachuzipus

Hallo Blinkly und herzlich [willkommenmr],

mal eine Teilantwort zum zweiten Ansatz:

> Bestimmen Sie die Lösung von
>  [mm]y' = (y + y^3) \cdot \sin(x), y(0)=1[/mm]
>  Lösen Sie die Dgl.
> sowohl als Typ "getrennte Veränderliche" wie auch als
> Bernoulli-DGL.
>  Hallo =)
>  
> Erst einmal vorweg:
>  Ich habe diese Frage in keinem Forum auf anderen
> Internetseiten gestellt ;-)
>  
> Habe die Aufgabe versucht zu lösen (auf beide Arten, mehr
> oder weniger erfolgreich) und habe leider noch ein paar
> Fragen, aber erstmal zu meiner Rechnung:
>  
> getrennte Veränderliche:
>  [mm]\int\frac{1}{y+y^3} dy = \int\sin(x)dx + c[/mm]
>  
> [mm]\int\frac{1}{y}dy - \int\frac{y}{y^2+1} dy = -\cos(x) + c[/mm]
>  
> [mm]\Rightarrow \ln|y| - \frac{ln|y^2+1|}{2} = -\cos(x) + c[/mm] [ok]

Ok soweit, nun Logarithmusgesetze bemühen: [mm] $\ln(a)-\ln(b)=\ln\left(\frac{a}{b}\right)$, [/mm] beachte auch: [mm] $\frac{1}{2}\ln(a)=\ln(\sqrt{a})$ [/mm]

Also [mm] $\ln\left(\frac{|y|}{\sqrt{y^2+1}}\right)=-\cos(x)+c$ [/mm]

e-Fkt. anwenden:

[mm] $\Rightarrow \frac{|y|}{\sqrt{y^2+1}}=e^{-\cos(x)+c}=\tilde c\cdot{}e^{-\cos(x)}$ [/mm] mit [mm] $\tilde c\in\IR^+$ [/mm]

Quadrieren:

Also [mm] $\frac{y^2}{y^2+1}=\hat c\cdot{}e^{-2\cos(x)}$ [/mm] mit [mm] $\hat c\in\IR^+$ [/mm]

Weiter: [mm] $y^2=\hat c\cdot{}e^{-2\cos(x)}\cdot{}y^2+\hat c\cdot{}e^{-2\cos(x)}$ [/mm]

Nun die [mm] $y^2$-Terme [/mm] auf eine Seite, ausklammern und dann $y$ auflösen.

Mit der AB das [mm] $\hat [/mm] c$ bestimmen ...


>  
> Und das nun nach y aufzulösen bereitet mir Probleme,
> sofern ich überhaupt richtig gerechnet hab bis hier hin
> ;-)
>  
> Möglicher Ansatz von mir:
>  [mm]\frac{1}{y^2} + y^2 + 1 = e^{2\cos(x)} - e^{2c}[/mm]
>  
> Mehr hab ich dazu leider noch nicht =/
>  
> Ich bedanke mich schon mal für Eure Mühe =)
>  
> Grüße
>  Blinkly


Gruß

schachuzipus

Bezug
                
Bezug
Bernoulli, getrennte Veränderl: Rückfrage
Status: (Frage) beantwortet Status 
Datum: 13:49 Mi 14.07.2010
Autor: Blinkly

Hallo schachuzipus :)

Erst einmal danke für Deine Antwort und dein herzliches Willkommen ;-)

Leider hab ich immer noch ein paar Fragen:
Zum einen bin ich mir noch nicht sicher, wann ich welches $c$ ersetzen darf (bspw. [mm] $\tilde [/mm] c$ und [mm] $\hat [/mm] c$), zum anderen versteh ich in diesem Zusammenhang nicht, warum bei Deiner Anwendung der e-Fkt. nun das $c$ multipliziert wird. Ich hoffe, das ist nicht eine zu doofe Frage =/

So, nun zu meiner Rechnung, das oben vorausgesetzt:
[mm] $y^2 [/mm] = [mm] \frac{e^{-2\cos(x)}\cdot \hat c}{1 - e^{-2\cos(x)}\cdot \hat c}$ [/mm]
$y = [mm] \sqrt{\frac{\hat c}{e^{2\cos(x)}-\hat c}}$ [/mm]
mit $y(0) = 1$ folgt dann:
[mm] $\hat [/mm] c = [mm] \frac{e^2}{2}$ [/mm]

Nun weiß ich nicht, ob ich das [mm] $\hat [/mm] c$ quasi wieder zurückrechnen muss, um es dann in die Gleichung einsetzten zu können. Ich habe leider auch noch kein Beispiel gefunden bisher, an dem ich das gut erkennen könnte.

Grüße
Blinkly

Bezug
                        
Bezug
Bernoulli, getrennte Veränderl: Antwort
Status: (Antwort) fertig Status 
Datum: 14:53 Mi 14.07.2010
Autor: MathePower

Hallo Blinkly,

> Hallo schachuzipus :)
>  
> Erst einmal danke für Deine Antwort und dein herzliches
> Willkommen ;-)
>  
> Leider hab ich immer noch ein paar Fragen:
>  Zum einen bin ich mir noch nicht sicher, wann ich welches
> [mm]c[/mm] ersetzen darf (bspw. [mm]\tilde c[/mm] und [mm]\hat c[/mm]), zum anderen
> versteh ich in diesem Zusammenhang nicht, warum bei Deiner
> Anwendung der e-Fkt. nun das [mm]c[/mm] multipliziert wird. Ich
> hoffe, das ist nicht eine zu doofe Frage =/


Nach der Integration erhältst Du

[mm]\ln\left(\frac{|y|}{\sqrt{y^2+1}}\right)=-\cos(x)+c[/mm]

Wendet man jetzt die e-Funktion darauf an, dann steht da:

[mm]e^{\ln\left(\frac{|y|}{\sqrt{y^2+1}}\right)}=e^{-\cos(x)+c}[/mm]

Da ln die Umkehrfunktion der e-Funktion ist, gilt:

[mm]\frac{|y|}{\sqrt{y^2+1}}=e^{-\cos(x)+c}[/mm]

Nach den Potenzgesetzen gilt:

[mm]e^{-\cos(x)+c}=e^{-\cos(x)}*e^{c}[/mm]

Dann definieren wir als neue Konstante

[mm]\tilde{c}:=e^{c}[/mm]

Mit der Definition dieser neuen Konstante ergibt sich:

[mm]\frac{|y|}{\sqrt{y^2+1}}=\tilde{c}*e^{-\cos(x)}, \ \tilde{c} \in \IR^{+}[/mm]

Jetzt werden beide Seiten quadriert:

[mm]\frac{y^{2}}{y^2+1}=\tilde{c}^{2}*e^{-2*\cos(x)}, \ \tilde{c} \in \IR^{+}[/mm]

Definieren wir hier wiederum als neue Konstante

[mm]\hat{c}:=\tilde{c}^{2}[/mm]

so ergibt sich:

[mm]\frac{y^{2}}{y^2+1}=\hat{c}*e^{-2*\cos(x)}, \ \hat{c} \in \IR^{+}[/mm]


>  
> So, nun zu meiner Rechnung, das oben vorausgesetzt:
>  [mm]y^2 = \frac{e^{-2\cos(x)}\cdot \hat c}{1 - e^{-2\cos(x)}\cdot \hat c}[/mm]
>  
> [mm]y = \sqrt{\frac{\hat c}{e^{2\cos(x)}-\hat c}}[/mm]


Eigentlich

[mm]y = \blue{\pm}\sqrt{\frac{\hat c}{e^{2\cos(x)}-\hat c}}[/mm]


>  mit [mm]y(0) = 1[/mm]
> folgt dann:
>  [mm]\hat c = \frac{e^2}{2}[/mm]


[ok]


>  
> Nun weiß ich nicht, ob ich das [mm]\hat c[/mm] quasi wieder
> zurückrechnen muss, um es dann in die Gleichung einsetzten
> zu können. Ich habe leider auch noch kein Beispiel
> gefunden bisher, an dem ich das gut erkennen könnte.


Nein, das [mm]\hat c[/mm] wird nicht zurückgerechnet.


>  
> Grüße
>  Blinkly


Gruss
MathePower

Bezug
                                
Bezug
Bernoulli, getrennte Veränderl: Mitteilung
Status: (Mitteilung) Reaktion unnötig Status 
Datum: 15:26 Mi 14.07.2010
Autor: Blinkly


> Nach der Integration erhältst Du
>  
> [mm]\ln\left(\frac{|y|}{\sqrt{y^2+1}}\right)=-\cos(x)+c[/mm]
>  
> Wendet man jetzt die e-Funktion darauf an, dann steht da:
>  
> [mm]e^{\ln\left(\frac{|y|}{\sqrt{y^2+1}}\right)}=e^{-\cos(x)+c}[/mm]
>  
> Da ln die Umkehrfunktion der e-Funktion ist, gilt:
>  
> [mm]\frac{|y|}{\sqrt{y^2+1}}=e^{-\cos(x)+c}[/mm]
>  
> Nach den Potenzgesetzen gilt:
>  
> [mm]e^{-\cos(x)+c}=e^{-\cos(x)}*e^{c}[/mm]

Danke! =)

War wieder mal etwas blind von mir, hätte aus der rechten Seite ein [mm] $e^{-\cos(x)} [/mm] + [mm] e^c$ [/mm] gemacht, was natürlich Quatsch ist ;-)
  

> Dann definieren wir als neue Konstante
>  
> [mm]\tilde{c}:=e^{c}[/mm]
>  
> Mit der Definition dieser neuen Konstante ergibt sich:
>  
> [mm]\frac{|y|}{\sqrt{y^2+1}}=\tilde{c}*e^{-\cos(x)}, \ \tilde{c} \in \IR^{+}[/mm]
>  
> Jetzt werden beide Seiten quadriert:
>  
> [mm]\frac{y^{2}}{y^2+1}=\tilde{c}^{2}*e^{-2*\cos(x)}, \ \tilde{c} \in \IR^{+}[/mm]
>  
> Definieren wir hier wiederum als neue Konstante
>  
> [mm]\hat{c}:=\tilde{c}^{2}[/mm]
>  
> so ergibt sich:
>  
> [mm]\frac{y^{2}}{y^2+1}=\hat{c}*e^{-2*\cos(x)}, \ \hat{c} \in \IR^{+}[/mm]

Super Erklärung, danke Dir =)


> > [mm]y = \sqrt{\frac{\hat c}{e^{2\cos(x)}-\hat c}}[/mm]
>  
>
> Eigentlich
>  
> [mm]y = \blue{\pm}\sqrt{\frac{\hat c}{e^{2\cos(x)}-\hat c}}[/mm]

Hoffentlich treten solche Unkonzentriertheiten in keiner Klausur auf ;-)
  

> Nein, das [mm]\hat c[/mm] wird nicht zurückgerechnet.

Macht auch beim 3. oder 4. Hinschauen keinen Sinn mehr, das [mm] $\hat [/mm] c$ zurück zu rechnen, da man es ja in eben diese Gleichung wieder einsetzt.

> Gruss
>  MathePower

Grüße
Blinkly

Bezug
        
Bezug
Bernoulli, getrennte Veränderl: Bernoulli
Status: (Antwort) fertig Status 
Datum: 15:10 Mi 14.07.2010
Autor: MathePower

Hallo Blinkly,

> Bestimmen Sie die Lösung von
>  [mm]y' = (y + y^3) \cdot \sin(x), y(0)=1[/mm]
>  Lösen Sie die Dgl.
> sowohl als Typ "getrennte Veränderliche" wie auch als
> Bernoulli-DGL.
>  Hallo =)
>  
> Erst einmal vorweg:
>  Ich habe diese Frage in keinem Forum auf anderen
> Internetseiten gestellt ;-)
>  
> Habe die Aufgabe versucht zu lösen (auf beide Arten, mehr
> oder weniger erfolgreich) und habe leider noch ein paar
> Fragen, aber erstmal zu meiner Rechnung:
>  
> Bernoulli:
>  [mm]y' = \sin(x)y + sin(x)y^3, \alpha = 3[/mm]
>  [mm]u = y^{-2}[/mm]
>  [mm]u' = -2\sin(x)u - 2\sin(x)[/mm]
>  
> [mm]u_h = e^{2\cos(x)}[/mm]
>  [mm]u_p = e^{2\cos(x)} \cdot \int \frac{-2\sin(x)}{e^{2\cos(x)}} dx = e^{2\cos(x)}\cdot -e^{-2\cos(x)} = -1[/mm]
> Substitution: [mm]t = \cos(x)[/mm]
>  [mm]\Rightarrow u = c \cdot e^{2\cos(x)} - 1[/mm]
>  
> [mm]y = \frac{1}{\sqrt{u}}[/mm]
>  [mm]y = \frac{1}{\sqrt{c\cdot e^{2\cos(x)}-1}}[/mm]
>  
> [mm]c = \frac{2}{e^2}[/mm] mit [mm]y(0) = 1[/mm]
>  [mm]\Rightarrow y = \frac{1}{\sqrt{2\cdot e^{2(\cos(x)-1)}-1}}[/mm]
>  
> Hier ist die Frage noch relativ einfach, stimmt das? :)


Ja. [ok]


>  
> Ich bedanke mich schon mal für Eure Mühe =)
>  
> Grüße
>  Blinkly


Gruss
MathePower

Bezug
                
Bezug
Bernoulli, getrennte Veränderl: Frage (beantwortet)
Status: (Frage) beantwortet Status 
Datum: 15:52 Mi 14.07.2010
Autor: Blinkly


>  >  [mm]\Rightarrow y = \frac{1}{\sqrt{2\cdot e^{2(\cos(x)-1)}-1}}[/mm]
>  
> >  

> > Hier ist die Frage noch relativ einfach, stimmt das? :)
>  
>
> Ja. [ok]
>  

Hihi, danke =)

Nun habe ich noch eine letzte, kurze Frage...

Ich habe ja nun zusammen mit Euch auf zwei verschiedenen Wegen diese Aufgabe gelöst und als Ergebnisse:

$y = [mm] \frac{1}{\sqrt{2\cdot e^{2(\cos(x)-1)}-1}}$ [/mm]

und

$y = [mm] \sqrt{\frac{\frac{e^2}{2}}{e^{2cosx}-\frac{e^2}{2}}} [/mm] = [mm] \frac{e}{\sqrt{2e^{2cos(x)}-e^2}}$ [/mm]

Müssten die gleich sein, bzw. sind sie das und ich sehe das einfach wieder nicht ?

Grüße
Blinkly

Bezug
                        
Bezug
Bernoulli, getrennte Veränderl: Antwort
Status: (Antwort) fertig Status 
Datum: 16:00 Mi 14.07.2010
Autor: Herby

Hi,

> >  >  [mm]\Rightarrow y = \frac{1}{\sqrt{2\cdot e^{2(\cos(x)-1)}-1}}[/mm]

>  
> >  

> > >  

> > > Hier ist die Frage noch relativ einfach, stimmt das? :)
>  >  
> >
> > Ja. [ok]
>  >  
>
> Hihi, danke =)
>  
> Nun habe ich noch eine letzte, kurze Frage...
>  
> Ich habe ja nun zusammen mit Euch auf zwei verschiedenen
> Wegen diese Aufgabe gelöst und als Ergebnisse:
>  
> [mm]y = \frac{1}{\sqrt{2\cdot e^{2(\cos(x)-1)}-1}}[/mm]
>  
> und
>  
> [mm]y = \sqrt{\frac{\frac{e^2}{2}}{e^{2cosx}-\frac{e^2}{2}}} = \frac{e}{\sqrt{2e^{2cos(x)}-e^2}}[/mm]
>  
> Müssten die gleich sein, bzw. sind sie das und ich sehe
> das einfach wieder nicht ?

Du kannst entweder den oberen Bruch mit e erweitern oder aus der Wurzel des unteren ein e herausziehen und anschließend kürzen.

LG
Herby

Bezug
                                
Bezug
Bernoulli, getrennte Veränderl: Mitteilung
Status: (Mitteilung) Reaktion unnötig Status 
Datum: 16:23 Mi 14.07.2010
Autor: Blinkly


> Du kannst entweder den oberen Bruch mit e erweitern oder
> aus der Wurzel des unteren ein e herausziehen und
> anschließend kürzen.
>  
> LG
>  Herby

Oh je, ich hab es extra ein paar mal hin und hergerechnet, aber ich glaube, es will heute nicht so...im Zweifelsfall schieb ich es auf die Wärme :P

Ich danke Dir für die schnelle Antwort auf diese blöde Frage und danke sowieso allen für ihre Hilfe =)

Grüße
Blinkly

Bezug
Ansicht: [ geschachtelt ] | ^ Forum "Gewöhnliche Differentialgleichungen"  | ^^ Alle Foren  | ^ Forenbaum  | Materialien


^ Seitenanfang ^
www.schulmatheforum.de
[ Startseite | Forum | Wissen | Kurse | Mitglieder | Team | Impressum ]